Betrag im C^n

Neue Frage »

lucio Auf diesen Beitrag antworten »
Betrag im C^n
Wie berechnet man denn den Betrag im C^n?

Was ist der Betrag von (1+i,2+3i,4,i) ??

Kann man C^n einfach als IR^2n auffassen wenns um den Betrag geht?

Also eigentlich soll ich zeigen, dass für Funktionen
g1,....gn: [a,b]--->C integrierbar und

sei definiert durch:



z.z:



gilt.
Abakus Auf diesen Beitrag antworten »
RE: Betrag im C^n
Zitat:
Original von lucio
Wie berechnet man denn den Betrag im C^n?

Was ist der Betrag von (1+i,2+3i,4,i) ??


Das hängt davon ab, was du unter einem "Betrag" verstehst oder anders gesagt, mit welcher Norm du den gerade betrachtest.

ZB wäre es möglich, die von dem üblichen Skalarprodukt induzierte Norm zu nehmen:



Der Querstrich oben bedeutet die komplexe Konjugation.

Grüße Abakus smile
lucio Auf diesen Beitrag antworten »

Hat jemand nen Tipp wie ich die Abschätzung denn zeigen kann?
Abakus Auf diesen Beitrag antworten »

Zitat:
Original von lucio
Hat jemand nen Tipp wie ich die Abschätzung denn zeigen kann?


Du könntest dir erstmal die Definition der obigen Integrale anschauen. Vielleicht findet sich von da an ja eine Abschätzung.

Grüße Abakus smile
ratskrone Auf diesen Beitrag antworten »

So ich habs mal umgeschrieben:




und



wie gehts nun weiter?
ratskrone Auf diesen Beitrag antworten »

Für Funktionen von [a,b]--->IR^n hab ich nun einen Beweis. Kann man nun sagen, dass es dann auch für C^n klappt wenn man C^n als IR^2n auffast? Oder muss man dazu noch was beweisen?
 
 
Abakus Auf diesen Beitrag antworten »

Zitat:
Original von ratskrone
Für Funktionen von [a,b]--->IR^n hab ich nun einen Beweis. Kann man nun sagen, dass es dann auch für C^n klappt wenn man C^n als IR^2n auffast? Oder muss man dazu noch was beweisen?


Da ich den Beweis nicht kenne, kann ich dazu nichts sagen. Meine Idee wäre den Beweis auf den Integralbegriff als Grenzwert zurückzuführen und dabei ggf. die Dreiecksungleichung zu verwenden.

Wie hast du es bewiesen ?

Grüße Abakus smile
ratskrone Auf diesen Beitrag antworten »
RE: Betrag im C^n
Ich hab das gezeigt aber nur für reellwertige Funktionen.




Dann hab ich geschrieben, dass das auch für komplexe gilt weil



gilt für
Abakus Auf diesen Beitrag antworten »
RE: Betrag im C^n
Zitat:
Original von ratskrone
Ich hab das gezeigt aber nur für reellwertige Funktionen.



Kannst du denselben Beweis auf übertragen ? Wie sieht der Beweis aus ? (Hast du hier mit Riemannschen Summen oder einfach mit der Positivität argumentiert ?)


Zitat:
Dann hab ich geschrieben, dass das auch für komplexe gilt weil



gilt für


Da würde ich nicht sehen, wie das folgen soll. Es mag sein, dass du das Argument anwenden kannst, bloß in der Kürze ist nicht zu erkennen, wie du das auf die Ungleichung mit den Integralen anwendest.

Grüße Abakus smile
ratskrone Auf diesen Beitrag antworten »

Der Beweis geht folgendermaßen:

setze

[latex]K^2=<u,u>=<\int_{b}^{a}~g(t)~dt,u>=\int_{b}^{a}~<g(t),u>dt\\ \leq \int_{b}^{a} ||g(t)||~||u||~dt=K~\int_{b}^{a}~||g(t)||~dt [latex]

Ich denk das kann man nicht so eifnach für C machen. Ich habs mir so gedacht man spaltet die einzelnen Funktionen in Real- und Imaginärteil auf. und erhält dann halt 2n rellwertige Funktionen.
ratskrone Auf diesen Beitrag antworten »

Der Beweis geht folgendermaßen:

setze



Ich denk das kann man nicht so eifnach für C machen. Ich habs mir so gedacht man spaltet die einzelnen Funktionen in Real- und Imaginärteil auf. und erhält dann halt 2n rellwertige Funktionen.
Abakus Auf diesen Beitrag antworten »
RE: Betrag im C^n
Zitat:
Original von lucio
Also eigentlich soll ich zeigen, dass für Funktionen
integrierbar gilt:



Ich fang mal vorne an. Du sollst die "Dreiecksungleichung für Integrale" zeigen. Ich gehe davon aus, dass hier R-Integrierbarkeit (R für Riemann) gemeint ist (möglich wären auch andere Integrierbarkeitsbegriffe, dann würde der Beweis entsprechend anders gehen).

Der erste Schritt ist nun, dass du zeigst, dass überhaupt R-integrierbar ist. Das ist nicht vorausgesetzt und auch nicht selbverständlich. Möglicherweise habt ihr das in einem Satz gemacht, den du zitieren kannst.

Der zweite Schritt besteht darin, die vorkommenden Integrale auf ihre Definition zurückzuspielen. Dazu kannst du dich hier informieren: Riemann-Integral.

Dazu betrachtest du Riemannsche Summen der Form:



ist deine Funktion, ist eine Zerlegung des Intervalls und ist eine Belegung.
( ist hier ein Vektor, summiert wird in jeder Komponente; die sonstigen Dinge solltest du ggf. nachlesen)

Nun gilt die Dreiecksungleichung für Vektoren:

Diese wird auf die Riemannsche Summe angewendet:



Wenn du nun entsprechende Riemann-Folgen bildest und zum Grenzwert übergehst (das darfst du wegen der R-Integrierbarkeit der entsprechenden Funktionen auf beiden Seiten: hier geht der erste Schritt ein), stehen auf beiden Seiten Integrale (genau die, die du haben willst).

Grüße Abakus smile

EDIT: Rechtschreibung
Neue Frage »
Antworten »



Verwandte Themen

Die Beliebtesten »
Die Größten »
Die Neuesten »